Загрузка страницы

Griffiths QM 6.13 (3d edition) Solution: Relativistic Correction for Quantum Harmonic Oscillator

In this video I will solve Problem 6.13 as it appears in the 3rd edition of Griffiths Introduction to Quantum Mechanics
The problem states:
Find the lowest order relativistic correction to the energy levels of the one-dimensional harmonic oscillator.

Here is Problem 2.37, which I refer to often throughout the video!
https://youtu.be/cE8bsRLdaPc
If you enjoy my content, please consider checking out my Patreon!
www.patreon.com/nickheumann

Also, consider subscribing and following me on my socials!

twitter: twitter.com/nickheumann
Instagram: instagram.com/nicolasheumann
twitch: www.twitch.tv/nickheumann

My name is Nick Heumann, I am a recently graduated physicist. I love to teach physics, so I decided to give YouTube a try. English is not my first language, but I hope that you can understand me well enough regardless.
▬ Contents of this video ▬▬▬▬▬▬▬▬▬▬
00:00 Introducing the Problem
01:17 Plugging everything into the General Formula
03:20 To find expectation value of X^2, check out problem 2.37 (link in description)
05:00 Finding the expectation value of x^4
08:00 Applying the raising and lowering operators to states n
20:10 Plugging in the result

Видео Griffiths QM 6.13 (3d edition) Solution: Relativistic Correction for Quantum Harmonic Oscillator канала Nick Heumann
Показать
Комментарии отсутствуют
Введите заголовок:

Введите адрес ссылки:

Введите адрес видео с YouTube:

Зарегистрируйтесь или войдите с
Информация о видео
12 июня 2022 г. 6:51:48
00:22:50
Другие видео канала
Griffiths Problem 4.4 (3rd ed) Solution: The OTHER Solution to the Angular EquationGriffiths Problem 4.4 (3rd ed) Solution: The OTHER Solution to the Angular EquationGriffiths QM Problem 4.8: Infinite Spherical Well for l=1Griffiths QM Problem 4.8: Infinite Spherical Well for l=1The EASIEST Explanation of the Gram-Schmidt Procedure (Griffiths QM 3rd ed. Problem 3.4)The EASIEST Explanation of the Gram-Schmidt Procedure (Griffiths QM 3rd ed. Problem 3.4)Griffiths Intro to QM - Problem 8.15: The ULTIMATE WKB Problem -HARDEST ONE YETGriffiths Intro to QM - Problem 8.15: The ULTIMATE WKB Problem -HARDEST ONE YETProving orthogonality and Completeness of Infinite Square Well WavefunctionsProving orthogonality and Completeness of Infinite Square Well WavefunctionsThe Integral You HAVE TO KNOW to MASTER Quantum Mechanics!The Integral You HAVE TO KNOW to MASTER Quantum Mechanics!How do LASERs work? A Quantum Mechanical Explanation Using Perturbation TheoryHow do LASERs work? A Quantum Mechanical Explanation Using Perturbation TheoryHow to determine the 4 Wavefunctions of the Hydrogen Atom for n=2How to determine the 4 Wavefunctions of the Hydrogen Atom for n=2How To Go From Arcsin(x) to Ln (natural logarithm) (Arcsin - Log Formulation)How To Go From Arcsin(x) to Ln (natural logarithm) (Arcsin - Log Formulation)Griffiths QM 6.26 (3rd ed) 6.28 (2nd ed): Hyperfine Splitting for Muonic H, Positronium, MuoniumGriffiths QM 6.26 (3rd ed) 6.28 (2nd ed): Hyperfine Splitting for Muonic H, Positronium, MuoniumGriffiths QM 2.27 Solution: Finding Transmission coefficient for double delta potentialGriffiths QM 2.27 Solution: Finding Transmission coefficient for double delta potentialThe WKB Approximation for Energy less than V - Finding the Transmission coefficientThe WKB Approximation for Energy less than V - Finding the Transmission coefficientWhy we need QFT & Derivation of Klein-Gordon Langriangian DensityWhy we need QFT & Derivation of Klein-Gordon Langriangian DensityHow to Solve The Infinite Spherical Well (FULLY EXPLAINED)How to Solve The Infinite Spherical Well (FULLY EXPLAINED)WKB Approximation Connection Formulas Example: Potential Well with no Vertical WallsWKB Approximation Connection Formulas Example: Potential Well with no Vertical WallsVariational Principle in QM Example 1: Quantum Harmonic OscillatorVariational Principle in QM Example 1: Quantum Harmonic OscillatorGriffiths QM 4.15 Solution: Proving Radial Wavefunction and finding expectation values for l=n-1Griffiths QM 4.15 Solution: Proving Radial Wavefunction and finding expectation values for l=n-1Solving the Schrödinger Equation for the Infinite Potential Well (Particle in a square box)Solving the Schrödinger Equation for the Infinite Potential Well (Particle in a square box)Griffiths QM Problem 2.37 (3rd edition): Find expectation values for nth stationary state of QHOGriffiths QM Problem 2.37 (3rd edition): Find expectation values for nth stationary state of QHOGriffiths Intro to QM 7.11 (3rd edition) solution: ground state energy of H using Ae^(-br^2)Griffiths Intro to QM 7.11 (3rd edition) solution: ground state energy of H using Ae^(-br^2)
Яндекс.Метрика